Taking the log of an exponential function and finding the slope

Click For Summary
The discussion revolves around solving a set of differential equations related to virus and T cell populations, specifically focusing on the function V(t) and its logarithmic representation. The key issue is the discrepancy in the expected slope of the log graph, which should be -μ, but is calculated as -μ/ln(10) due to the use of logarithm base 10 in the problem. The participant is unsure whether to interpret "log" as the natural logarithm or base 10, given the context of the problem and the notation used in the textbook. They highlight that the problem's requirement to determine μ from the slope of the log graph complicates the situation. Ultimately, clarity on the logarithmic base is essential for correctly interpreting the results.
cryora
Messages
49
Reaction score
3
This is part of a differential equations group project problem where I solve a set of differential equations to obtain the solution to a function. The part that I am stuck at involves taking the log of an exponential function, though there may be a mistake on the book's part, but I'm not sure.

Homework Statement


Argue from your formula for V(t), that the graph of V(t) on a log scale (i.e., the graph of log V) over an extended period of time (say, several weeks) will tend toward a graph of a straight line whose slope is either -γ (the negative reciperocol of the average lifespan of a free virus) or -μ (the negative reciprocol of the average lifespan of an infected CD4+ T cell), according to whether γ or μ is smaller.

Homework Equations


\frac{d}{dt}T(t) = λ - δT(t) - βV(t)T(t)
\frac{d}{dt}I(t) = βV(t)T(t) - μI(t)
\frac{d}{dt}V(t) = NμI(t) - γV(t)
T represents population of uninfected T cells in units of cells.
I represents population of infected T cells in units of cells.
V represents population of virus in units of virions.
λ is the rate of T cell production by the human body per day in units of 1/days
δ is the rate constant of T cells naturally dying off per day in units of 1/days
μ is the rate constant of Infected T cells dying off (bursting) per day resulting in the spread of virus in units of 1/days
γ is the rate constant of virus decaying per day in units of 1/days
N is the number of virus per cell in units of virions/cell
β is the infection rate constant in units of 1/(days*virions)

The problem states to set β = 0, assuming there is a drug that completely removes infection of T cells, allowing me to solve by 1st order linear differential equation methods to get V(t):
V(t) = \frac{NμI_0}{γ-μ}e^{-μt} + (V_0 - \frac{NμI_0}{γ-μ})e^{-γt}
with initial conditions
V(0) = V_0 \text{ and } I(0) = I_0

The Attempt at a Solution


Taking the log of V(t) and then taking the derivative, I get:
\frac{d}{dt}log[V(t)] = \frac{d}{dt}\frac{ln[V(t)]}{ln(10)} = \frac{1}{ln(10)V(t)}\frac{d}{dt}V(t)
and
\frac{d}{dt}V(t) = \frac{-Nμ^2I_0}{γ-μ}e^{-μt} - γ(V_0 - \frac{NμI_0}{γ-μ})e^{-γt}

Assuming μ is smaller than γ, the term containing e^{-γt} should go to 0 when t is large, leaving us with:
\frac{d}{dt}log[V(t)] = \frac{V'(t)}{ln(10)V(t)} = \frac{\frac{-Nμ^2I_0}{γ-μ}e^{-μt}}{ln(10)\frac{NμI_0}{γ-μ}e^{-μt}} = \frac{-μ}{ln(10)} \text{for large t}

The problem is the factor of \frac{1}{ln(10)} causing the slope of the log graph to be inconsistent with what the problem statement claims it should be: -μ.
 
Last edited:
Physics news on Phys.org
When doing calculus, "log" means "natural logarithm".
 
pasmith said:
When doing calculus, "log" means "natural logarithm".

While that would get rid of the 1/ln10 factor, I doubt I could make that assumption in this case because for one this book does use ln notation, and there are graphs for this problem that depicts virus amount in a log base 10 scale (where it goes from 1 to 10 to 100 to... 10^n... every major gridline). The problem even states to determine μ based on the slope of those graphs.
 
Question: A clock's minute hand has length 4 and its hour hand has length 3. What is the distance between the tips at the moment when it is increasing most rapidly?(Putnam Exam Question) Answer: Making assumption that both the hands moves at constant angular velocities, the answer is ## \sqrt{7} .## But don't you think this assumption is somewhat doubtful and wrong?

Similar threads

Replies
7
Views
2K
Replies
2
Views
2K
  • · Replies 4 ·
Replies
4
Views
1K
  • · Replies 2 ·
Replies
2
Views
2K
  • · Replies 2 ·
Replies
2
Views
2K
Replies
4
Views
2K
  • · Replies 1 ·
Replies
1
Views
1K
  • · Replies 7 ·
Replies
7
Views
16K
  • · Replies 1 ·
Replies
1
Views
1K
  • · Replies 3 ·
Replies
3
Views
3K